LSAT and Law School Admissions Forum

Get expert LSAT preparation and law school admissions advice from PowerScore Test Preparation.

User avatar
 Dave Killoran
PowerScore Staff
  • PowerScore Staff
  • Posts: 5852
  • Joined: Mar 25, 2011
|
#73084
Complete Question Explanation
(The complete setup for this game can be found here: https://forum.powerscore.com/lsat/viewtopic.php?t=12575)

The correct answer choice is (A)

The question stem specifies that the lists are of unsliced loaves. From our discussion of the rules, the third, fourth, and fifth rules address unsliced loaves, and thus you should look to one of those rules to produce the correct answer in this Could Be True Except question (which is identical to Cannot Be True).

Answer choice (A) cannot occur because from the fifth rule when there are two or more unsliced loaves, at least one must be rye. Thus, answer choice (A) is correct.

Get the most out of your LSAT Prep Plus subscription.

Analyze and track your performance with our Testing and Analytics Package.